0 Daumen
434 Aufrufe

Leider ist mein Ergebnis falsch! Wo liegt mein Fehler?

Lösungsweg:

M(t)= K(t)/t = 0,004t+7010/t+4

K'(t)= 0,004-7010t2=0

t= 5.295,3Bild Mathematik

Avatar von

1 Antwort

+1 Daumen

K'(t)= 0.004-7010/t2=0

Wolfram gibt dann 

t≈1324 h

Avatar von 37 k

Leider verstehe ich nicht wie ich auf die Zahl komme.

Die Gleichung löse ich ja folgendermaßen auf:

K'(t)=0,004-7010/t2=0          Ι -0,004

K'(t)=-7010/t2=-0,004           Ι *(-7010)

t2= 28,04                              Ι √

t= 5.295,30

Wenn vor dem t^2 die Zahl 7010 als Faktor steht, dann musst du durch diese Zahl teilen:

0.004-7010/t2=0

-7010/t^2=-0.004 | *(-1)

7010/t^2=0.004 | :7010

1/t^2≈ 5.706 *10^{-7} | Kehrwert bilden

t^2=1752500 | √

t ≈ 1324 (die negative Lösung entfällt)


Jetzt verstehe ich!
Lieben Dank für die Mühe :)

Ein anderes Problem?

Stell deine Frage

Willkommen bei der Mathelounge! Stell deine Frage einfach und kostenlos

x
Made by a lovely community